A pack of gum cost $0.80. If the tax is an additional 5%, what is the total INCLUDING
tax?

Answers

Answer 1

Answer:

$0.84 I think is the answer


Related Questions

Please help 50 points

What is the fractional form of

Answers

Answer:

it's option A .............

Answer:

8/9

Step-by-step explanation:

Repeating. Just do 8 divide by 9

Help I give crownnnn !!!

Answers

Answer:

it is 11

Step-by-step explanation:

i did the math

Answer:

11

Step-by-step explanation:

1x2=2

3x3=9

9+2= 11

I need help please help me , I don’t know the answer

Answers

Answer:

b = 4 inches, c = ~5.66 inches

Step-by-step explanation:

The triangle is isosceles so b is equal to the adjacent side. Use the Pythagorean theorem to find c.

HELP ME WITH THIS ONE PLASE TYYY

Answers

Answer b

Step-by-step explanation:

Because Adjacent angles are two angles that have a common vertex and a common side but do not overlap

Help please I will give you the brainliest

Answers

Answer:

x = 125

y = 55

Step-by-step explanation:

opposite angles in a parallelogram are equal

we also know that this is true because

(2(125)) + (2(55)) = 360

There are 9 computers and 72 students. What is the unit rate of students to computers? A) 1 computer 8 students B) 8 students 1 computer C) 3 computers 24 students D) 24 students 3 computers

Answers

Answer:

72/9 = 8

Answer is b

8 students to 1 computer

Just 4 people who need helppp

Answers

Answer:

1/2 ÷ 4 = 8

Step-by-step explanation:

Dividing fractions is as easy as pie. Flip the second and multiply.

1/2 ÷ 4 = 1/8

A whole number secretly has a 1 underneath, like this: 4/1

Flip the second number over to get 1/4, then multiply straight across.

[tex]\frac{1}{2}[/tex] × [tex]\frac{1}{4}[/tex] = [tex]\frac{1}{8}[/tex]

Try to do the same for the other answers and see if they're right (they're not).

1/8 x 1/6 = 1/48

1/3 x 1/4 = 1/12

1/9 x 1/2 = 1/18

Find the area of the shape shown below.

Answers

Answer:

42

Step-by-step explanation:

to find out the triangle do this formula:

bxh/2

3.5x3=10.5

3.5x3=10.5

21

To find out rectangle do:

lxw

7x3=21

21+21=42

what is the anwser -13-4x=x+7

Answers

-13-4x=x+7

+13         +13

-4x=x+20

-x    -x

-5x=20

/-5   /-5

x=-4

---

hope it helps

Can someone pls help me with this?

Answers

Answer:

the answer is d

Step-by-step explanation:

so for one do y-intercept has to be at (0,25) so we can eliminate answer choices B and C

the reason why it is not a is because a is moving out of constant rate of 25 when it needs to be moving at a constant rate like 10 D is the answer because it's moving at a constant rate of 10 and it has a y intercept of (0,25)

The answer is graph D

It starts out at 25 and each week the graph goes up only 10 dollars, so it’s the only one that makes sense. Good luck!!!

ITS AN EMERGENCY PLEASE HELPP MEEEE !!!!

Answers

Answer: 832

Step-by-step explanation:

Does anyone know this answer?

Answers

Answer:

The answer is Vertical

Step-by-step explanation:

Hope this helps <3

Answers? Thank you for helping

Answers

Answer:

1. no solution

2. one solution

3. no solution

4. no solution

Step-by-step explanation:

Please help me with a division problem. 97/4=

Answers

the answer is ~ 24.25
That’s easy math the answer to this is


24 with a remainder of 25. To

Can someone pls give me an explanation on how to do this pls

Answers

Answer: This is a perfect square trinomial. You are trying to find the binomials that multiply to that. In order to do that, you do the following.

Multiply 2x^2 by 4 (A times C)

Find what numbers multiply to give you 8 and add to give you 7.

When you do this, you will notice that there are no numbers that do this. That makes your answer D. NONE OF THESE.

Step-by-step explanation:

Answer:

J

Step-by-step explanation:

F is impossible because -1 x 4 = -4

G is impossible because 1 x -4 = -4

Try to solve H

2x^2+8x+x+4 = 2x^2+9x +4 (is not correct)

If x+1/x= 3, then prove that m^5+1/m^5= 123

Answers

9514 1404 393

Explanation:

We can start with the relations ...

  [tex]\displaystyle\left(x+\frac{1}{x}\right)^3=\left(x^3+\frac{1}{x^3}\right)+3\left(x+\frac{1}{x}\right)\\\\\left(x+\frac{1}{x}\right)^5=\left(x^5+\frac{1}{x^5}\right)+5\left(x^3+\frac{1}{x^3}\right)+10\left(x+\frac{1}{x}\right)\\\\\textsf{From these, we can derive ...}\\\\x^5+\frac{1}{x^5}=\left(x+\frac{1}{x}\right)^5-5\left(\left(x+\frac{1}{x}\right)^3-3\left(x+\frac{1}{x}\right)\right)-10\left(x+\frac{1}{x}\right)[/tex]

  [tex]\displaystyle x^5+\frac{1}{x^5}=\left(x+\frac{1}{x}\right)^5-5\left(x+\frac{1}{x}\right)^3+5\left(x+\frac{1}{x}\right)\right)\\\\x^5+\frac{1}{x^5}=3^5 -5(3^3)+5(3)\\\\=((3^2-5)3^2+5)\cdot3=(4\cdot9+5)\cdot3=(41)(3)\\\\=\boxed{123}[/tex]

A triangular prism is shown below.
5 cm
14 cm
3 cm
8 cm
A formula for the volume of a triangular prism is V= Bh. Which expression can be used to find V
the volume of the prism in cubic centimeters?

Answers

Answer:

(5)(8)(14)

Step-by-step explanation:

Helppp Meee Plsss
....

Answers

Answer:

I believe it is because it is being transferred to kinetic energy

Step-by-step explanation:

As the puck falls, it loses potential energy and gains kinetic energy because it is now falling, which causes movement due to gravity. And as we know, kinetic energy is energy which a body possesses by virtue of being in motion

Pls I need help
Hahaha I did not reach the word limit

Answers

Answer:

5-(-7)=5+7

Step-by-step explanation:

keep in mind that two negatives make a positive. therefore it would be 5+7=5+7. i hope this helps :)

Simplify the sum.
(2u3 + 6u2 + 3) + (2u3 – 7u + 6)
A.
9-74 + 642 + 403
03 + 62 - 7 + 9
Ou3 - 742 + 60-9
403 + 642 - 74 + 9

Answers

Answer:

4u^3 + 6u^2 - 7u + 9

Step-by-step explanation:

(2u3 + 6u2 + 3) + (2u3 – 7u + 6)

Remove the brackets.

2u^3 + 6u^2 + 3 + 2u^3 - 7u + 6

2u^3 + 2u^3 = 4u^3

6u^2

- 7u

3 + 6 = 9

Now just read down

4u^3 + 6u^2 - 7u + 9

The answers don't seem to connect with the question. Is U equal to something?

PLS HELPPPPPPPPPPPPPPPPPPPPPPPPP

Answers

Answer:

(-64)[tex]^{-\frac{1}{3}[/tex]

Step-by-step explanation:

Answer:

(-64)^-5/3 X (-64)^4/3 = -0.25 or -1/4.

Step-by-step explanation:

l hope that this helps! :))>

9,524 divided by 126

Answers

Answer:

75.5

Step-by-step explanation:

9,524 divided by 126 is equal to 75 with a remainder of 30.

What is an expression?

An expression contains one or more terms with addition, subtraction, multiplication, and division.

We always combine the like terms in an expression when we simplify.

We also keep all the like terms on one side of the expression if we are dealing with two sides of an expression.

Example:

1 + 3x + 4y = 7 is an expression.com

3 + 4 is an expression.

2 x 4 + 6 x 7 – 9 is an expression.

33 + 77 – 88 is an expression.

We have,

To perform the division of 9,524 by 126, you can use long division as follows:

     

 126 | 9,524 | 75

      882

      ---

      1124

     1008

     ----

      1164

     1134

     ----

        30

In other words:

9,524 ÷ 126 = 75 R 30

Therefore,

9,524 divided by 126 is equal to 75 with a remainder of 30.

Learn more about expressions here:

https://brainly.com/question/3118662

#SPJ2

What is the value of x in the figure below?

A-9
B-14 1/3
C-45 2/3
D-81

Answers

Answer:

A

Step-by-step explanation:

3x9+ 27 + 8= 35

What is the present value of an ordinary annuity having semi-annual payments of 8000 for 12 years with an interest rate of 12%compound semi- annually?

Answers

Answer:

PV= $100,402.86

Step-by-step explanation:

To calculate the present value, first, we need to determine the future value using the following formula:

FV= {A*[(1+i)^n-1]}/i

A= semiannual deposit

n=12*2= 24 semesters

i= 0.12/2= 0.06

FV= {8,000*[(1.06^24) - 1]} /  0.06

FV= $406,524.62

Now, the present value:

PV= FV / (1+i)^n

PV= 406,524.62 / 1.06^24

PV= $100,402.86

i need to know what type of angle that is the answer choices are plzzz help me plzz

1.supplementary angles

2.complementry angles

3.verticles angles

4.obtuse angles

Answers

They are complementary angles. Angles that add up to 90°

solve for 65+5x and show work !!!

Answers

Answer:

x = -13

Step-by-step explanation:

65 + 5x = 0

65 = -5x

divide by -5

-13 = x

Answer:

x = 13

Step-by-step explanation:

You have to get x by itself

so you divide 5 by 65

Then X is by itself and your answer is 5/65

Please help with geometry work asap!!!

Answers

Answer:

z = 100°

x = 40°

Step-by-step explanation:

Triangle CBD is isosceles so there are two 50°

DBC = 50°

That means BDC = 80° because 180(sum of angles in triangle) - 50 - 50 = 80

Z = 100° bc angle of straight line is 180° and 180 - 80 = 100

Now triangle ADB is also isosceles because there are two sides of equal length

So there will be 2*x + 100 = 180

x = 40°

Pls help me it’s due hellop

Answers

Answer:

m<CAB = 59

m<ACB = 59

Step-by-step explanation:

4x - 41 = 2x + 9

2x - 41 = 9

2x = 50

x = 25

m<CAB: 4x - 41

4(25) - 41

59

m<ACB: 2x + 9

2(25) + 9

59

where is the code for genes stored

Answers

The genes are stored on two strands of a DNA molecules as a linear, non-overlapping sequence of the nitrogenous bases Adenine (A), Guanine (G), Cytosine (C) and Thymine (T). These are the "alphabet" of letters that are used to write the "code words".

Hope it helped

f(2)if f(x)=2x-5 plz answer

Answers

Answer:

-1

you use the given functions to set up and simplify f(2)

Other Questions
5 16. Your checking account had a starting balance of $623.45. You make a withdraw at an ATM for $50 and are charged a $3 fee. You deposit a check from your mom for your birthday for $75 and then use your debit card for $7.29 at Dutch Bros. You write a check to T-Mobile for $36.92 for your monthly bill. What is your current balance? A coin is flipped 20 times. The results are 12 heads and 8 tails. Determine the theoretical probability of getting heads.60%40%20%50% Select all the correct answers.Which two sentences about trade in the Roman Republic are true?1. People exchanged goods instead of using money to buy them.2. The Romans used gold, silver, and bronze coins for trade and to pay taxes.3. Goods came to the Roman Republic from several regions outside of Italy.4. Roman soldiers were paid with goods from all over the region. Is the following claim supported by a strong or weak evidence? How do you know? Explain in 3 to 4 sentences.Schools should teach students the skill of active listening so they can be successful in the workplace. Active listening is just as important to talking. Adults spend 70 percent of their time communicating and, of that, 45 percent of their time is listening. Studies have shown that employers are more likely to hire applicants with good communication skills. The two dot plots represent a sample of the number of people in households in two towns. Which statements are true about the data sets?Check all that apply.Both have the same number of data points.Both means are between 3 and 4.Both have the same median.Both have the same range.Westwood has less variability than Middleton. Polygon ABCD has the following vertices:A(4, 2), B(3, 2), C(3, 5), and D(4, 2)Calculate the area of the polygon. Answers:A) 28 units squaredB) 38.5 units squaredC) 40.5 units squaredD) 49 units squared What is the basic level of an ecosystem? What is 2+2?If you answer correctly you will get a brainiest Help will mark brainliest A bag contains 14 blue marbles, 10 red marbles, and 6 yellow marbles.If Henna randomly draws a marble from the bag (and puts it back) 25 times, how manytimes should she expect to pull a yellow marble? All other organisms in a community are consumers, also called:O a. Carcassesb. Heterotrophs. What country was Judaism founded in What type of fruit do good and bad trees produce? (Refering to the bible) During the current year, Ron and Anne sold the following assets: Capital Asset Market Value Tax Basis Holding Period L stock $50,000 $41,000 > 1 year M stock 28,000 39,000 > 1 year N stock 30,000 22,000 < 1 year O stock 26,000 33,000 < 1 year Antiques 7,000 4,000 < 1 year Rental home 350,000* 90,000 > 1 year *$30,000 of the gain is 25 percent gain (from accumulated depreciation on the property).Ignore the Net Investment Income Tax.Given that Ron and Anne have a taxable income of $400,000 (all ordinary) before considering the tax effect of their asset sales, what is their gross tax liability for 2016 assuming they file a joint return? who is first prime minister of malaysia? what is the value of x? SOMEONE PLS PLSPLS HELP ME!!give three examples of the types of materials that would be used whenbuilding houses where their thermal conductivity would need to beconsidered. HelpCassius is angry because Brutus condemned a man who took bribes, despite the fact that Cassiustold him not to. Brutus, ever concerned with honesty, is angry because he also believes that Cassiushas himself been taking bribes. For Brutus, such immorality would completely defeat the purpose ofhaving killed Julius Caesar; in his mind, he wonders why they would kill a corrupt man only tobecome corrupt themselves.Refer to Brutus's dialogue through line 17. What is his intent? A. To accuse Cassius of being hypocritical by saying one thing and doing another.B. To highlight Cassius's wartime qualifications.C. To express admiration for qualities he does not have, but Cassius does.D. To inform Cassius of his thoughts on the upcoming battle. 1. What part of the world originally had the most forest land? 400x60x80x678x56x45x5435